Top Banner
TEST 21 1 A 23-year-old professional basketball player goes to the team physician 3 hours before game time complaining of abdominal pain. The symptoms began approximately 8 hours earlier in a diffuse fashion. Two hours later, he began feeling nauseated and vomited twice. Over the past 4 hours, the abdominal pain has become more severe and well localized in the right lower quadrant. His examination now reveals well-localized pain in the right lower quadrant inferolateral to the umbilicus. Which of the following is the most likely diagnosis? A. Acute obstruction of the appendiceal lumen by a fecalith B. Acute onset of ileocolitis C. Acute onset of ischemic colitis D. Acute Yersinia infection E. Obstruction of the ileocecal valve by a mass Explanation: The correct answer is A. Acute appendicitis is the second most common cause in the U.S., behind hernia, of severe acute abdominal pain that requires abdominal operation. Although it
95
Welcome message from author
This document is posted to help you gain knowledge. Please leave a comment to let me know what you think about it! Share it to your friends and learn new things together.
Transcript
Page 1: TEST 21

TEST 211A 23-year-old professional basketball player goes to the team physician3 hours before game time complaining of abdominal pain. The symptoms beganapproximately 8 hours earlier in a diffuse fashion. Two hours later, he beganfeeling nauseated and vomited twice. Over the past 4 hours, the abdominalpain has become more severe and well localized in the right lower quadrant.His examination now reveals well-localized pain in the right lower quadrantinferolateral to the umbilicus. Which of the following is the most likely diagnosis?

  A. Acute obstruction of the appendiceal lumenby a fecalith

  B. Acute onset of ileocolitis

  C. Acute onset of ischemic colitis

  D. Acute Yersinia infection

  E. Obstruction of the ileocecal valve by amassExplanation:The correct answer is A. Acuteappendicitis is the second most common cause in the U.S., behind hernia, ofsevere acute abdominal pain that requires abdominal operation. Although itcan occur at all ages, many patients, like this man, are teenagers or youngadults. This patient's presentation is typical for acute appendicitis, withinitially poorly localized pain that is followed by nausea and vomiting. Inclassic appendicitis, the pain shifts to the right lower quadrant, where itbecomes more localized. In most patients, acute obstruction of the appendicealorifice by a fecalith initiates the acute appendicitis.The acute onset of ileocolitis (choiceB) will produce diarrhea or bloody stools.There is no evidence to suggest an etiology for ischemic colitis (choice C), which will typically present withbloody diarrhea and often with left-sided abdominal pain.Acute Yersinia infection (choice D) will produce acute right lowerquadrant findings similar to those of acute appendicitis. However, it is accompaniedby diarrhea, which is not described in this case.There is no reason to suspect obstruction of the ileocecal area by anymass (choice E) in a 23-year-old

Page 2: TEST 21

man. Such an obstruction, should it occur, would typically present with abdominal distention as a result of bowel obstruction.2A man who appears to be in his twenties is brought to the emergencydepartment by the police after being found wandering around a local airport.The patient claims not to know how he came to this city and states repeatedlythat everything that he sees in the environment is unfamiliar. He has no identificationor baggage and cannot remember his name. He is well groomed and in no apparent distress. His process of thought is logical and he denies any mood complaintsor perceptual disturbances. Physical examination, metabolic screen and toxicologies,neuroimaging, and electroencephalography are all within normal limits. Which of the following is the most likely diagnosis?

  A. Adjustmentdisorder

  B. Bipolardisorder, most recent episode manic

  C. Dissociativefugue

  D. Dissociativeidentity disorder

  E. Major depressivedisorderExplanation:The correct answer is C. Dissociativefugue is a rare dissociative disorder involving abrupt travel and an amnesiafor events that may include the circumstances preceding arrival at one's destination and one's identity. The etiology of dissociative fugue is unknown, thougha possible correlation between dissociative fugue and recent life stressorsmay exist. The condition is usually self-limited. Adjustment disorder (choice A)is characterized by maladaptive reactions to life stressors that are thoughtto be in excess given the nature and the severity of the stressor. Frank dissociationand fugue are in excess of what could be considered in the adjustment disorderspectrum. Travel and spending money can be components of a manic episode (choice B). However, a failure to remember

Page 3: TEST 21

one's identity and most recent events is not a feature of mania. Additionally,other neurovegetative symptoms of mania such as irritability, psychomotoragitation, and pressured speech are not evident in this patient. Dissociative identity disorder (choiceD) is a dissociative condition that is defined by the presence of multiple discrete personalities that manifest in the same individual. Thispatient does not demonstrate evidence of multiple distinct personalities. There is nothing in the patient's history to indicate a major depressiveepisode (choice E). Findings suchas prominent depressed mood, tearfulness, weight loss, or suicidal ideationare not evident in this patient.3A 21-year-old woman comes to the physician because of abdominal pain.She states that the pain is in her right lower quadrant and has been gettingworse over the past 3 months. She has no other symptoms and a normal appetite.Examination demonstrates mild right lower quadrant abdominal tenderness. Pelvicexamination reveals mild right adnexal enlargement and tenderness. Urine human chorionic gonadotropin (hCG) is negative. A pelvic ultrasound is obtainedthat shows a 4-centimeter, heterogeneous hyperechoic lesion in the right adnexawith cystic areas. On transvaginal ultrasound, hair and calcifications aredemonstrated within the cystic areas. Which of the following is the most likely diagnosis?

  A. Appendicitis

  B. Benigncystic teratoma (dermoid)

  C. Corpus luteumcyst

  D. Ectopicpregnancy

  E. Tubo-ovarianabscessExplanation:The correct answer is B. Thispatient has a presentation and findings that are most consistent with a benigncystic teratoma (dermoid). Cystic teratomas are, by far, the most common typeof ovarian neoplasm: cystic teratomas account for 25 to 40% of all ovarian

Page 4: TEST 21

neoplasms. They are a type of ovarian germ cell tumor, which can range insize from small masses that are noted incidentally on ultrasound and causeno symptoms to larger cysts that cause pain and pressure, as this patienthas. A single germ cell gives rise to a teratoma. Because the germ cell istotipotential, the dermoid is characterized by all three germ cell layers:ectoderm, mesoderm, and endoderm. Gross examination of a dermoid will oftenreveal skin, bones, hair, and teeth, which can often be seen on ultrasound.The part of the dermoid that contains the largest number of different tissuesis called Rokitansky's protuberance. Laparotomy is usually the most appropriatemanagement of a patient with a dermoid because, as adnexal masses enlarge,the risk of ovarian torsion increases. Also, dermoids may cause symptoms ofpain and pressure and, on that basis, should be removed. At the time of surgery, close examination should be made of the other ovary because dermoids may befound bilaterally in more than 10% of cases.Appendicitis (choice A) isusually not a chronic process slowly developing over 3 months. Also, patientswith appendicitis typically have anorexia and appear ill.A corpus luteum cyst (choice C)is a common cause of a complex adnexal mass in a young woman. However, thispatient has a presentation and a mass with ultrasound characteristics thatare classic for dermoid.Ectopic pregnancy (choice D)should always be considered when a woman of childbearing age presents withabdominal pain. A negative urine hCG effectively rules out ectopic pregnancy.Patients with a tubo-ovarian abscess (choice E) usually have fevers, significant abdominal and pelvictenderness, and appear ill.4A 33-year-old woman is undergoing a diagnostic work-up because she appearsto have Cushing syndrome. She has elevated levels of cortisol, which are notsuppressed when she is given high-dose dexamethasone. ACTH levels are greaterthan 200 pg/ mL. A chest x-ray film shows a central, 3-cm round mass on thehilum of the right lung. Bronchoscopy and biopsies confirm a diagnosis ofsmall cell carcinoma of the lung. Which of the following is the preferred treatment for this woman?

  A. Bilateraladrenalectomy

  B. Generalsupport only

  C. Pneumonectomy

Page 5: TEST 21

  D. Radiationand chemotherapy directed at the lung cancer

  E. Trans-sphenoidalhypophysectomy and pulmonary lobectomyExplanation:The correct answer is D. Theendocrine workup is indicative of ectopic ACTH production, and the obvioussite is small cell carcinoma of the lung. The lung cancer is what is goingto kill this woman, not the endocrine manifestations of the tumor. Althoughsmall cell carcinoma of the lung is rarely cured, longer survival can be obtainedwith radiation and chemotherapy.Adrenalectomy (choice A) wouldaddress the endocrine problem by depriving the ectopic ACTH of its targetgland. But, as pointed out above, the lethal disease here is the lung cancer.General support only (choice B)would lead to death in about 2 months. Radiation and chemotherapy can prolongsurvival by approximately 2 years.Pneumonectomy (choice C) isnot the treatment for small cell carcinoma of the lung. It is the appropriatetreatment for resectable and potentially curable non?small cell cancers ofthe lung.The combination of hypophysectomy and lobectomy (choice E) is wrong for several reasons. Pituitary microadenomassuppress with high-dose dexamethasone, and their production of ACTH is typicallymuch lower than that seen in ectopic disease (i.e., <200 pg/mL). Thus,this woman does not need pituitary surgery. As far as the lung is concerned,surgery is not the treatment for small cell carcinoma, and lobectomy wouldnot be applicable for a central, hilar tumor.5A 33-year-old woman without any medical history comes to the emergencydepartment complaining of the worst headache of her life. The headache occurredsuddenly and awoke her from sleep approximately 32 hours before presentation.She delayed coming to the hospital and was trying to manage the pain at homewith ibuprofen and acetaminophen. Further history reveals that her mother, grandfather, and three sisters all have kidney disease, and her mother andone sister are currently on hemodialysis. Her aunt died from a bleed in herhead when she was 40 years of age and had kidney problems also. Which of the following studies is the most sensitive test to diagnosisthis patient's acute problem?

  A. Bleeding

Page 6: TEST 21

scan

  B. CT scanof the head

  C. Genetictesting

  D. Lumbarpuncture

  E. MRI of theheadExplanation:The correct answer is D. Thispatient is having a subarachnoid bleed. Clues to the diagnosis are the acuteonset of "the worst headache of my life." Her family history of kidney diseaseshould suggest polycystic kidney disease, which is associated with aneurysmformation in the Circle of Willis. Aneurysm formation is the most common etiologyfor subarachnoid bleeds. The most sensitive diagnostic test is the lumbarpuncture. The finding you would expect with a subarachnoid hemorrhage is ayellowing of the fluid from the hemolysis of red blood cells. This is knownas xanthochromia. Bleeding scan (choice A) isa test in which red blood cells are "tagged" and injected into a patient to find an area of bleeding. They are often used in the evaluation of gastrointestinalbleeding but do not have a regular role in the routine workup of subarachnoidhemorrhage. CT of the head (choice B)is the first test that is done in the evaluation of a bleed. Its sensitivityis approximately 95% early on, but slowly decreases after 24 hours after onsetof the bleeding. If the CT scan is negative but bleeding is still suspected,you would go on to the lumbar puncture, which is the most sensitive test forsubarachnoid hemorrhage. Genetic testing (choice C)would likely reveal that this patient has polycystic kidney disease. Althoughit is important for our patient to know this information, at the present timeit is more important to diagnose any intracranial process. MRI of the head (choice E)is less sensitive than a CAT scan for finding bleeds. It is also more expensive,more time consuming, and less readily available. It is not the proper testfor this patient.

Page 7: TEST 21

6A 69-year-old woman comes to her physician's office complaining of 1week of crampy lower abdominal pain and bloody diarrhea. She had previouslybeen followed for symptoms of stable exertional angina and hypertension. Shehad an uncomplicated myocardial infarction 3 years earlier. Her symptoms began1 week ago with mild postprandial abdominal cramping followed by diarrhea, which became bloody after 2 days. She has not traveled recently and is a retiredlibrarian. Her temperature is 38.6 C (101.4 F), blood pressure is 120/84 mmHg, and pulse is 96/min. She has moderate tenderness to palpation of the leftlower quadrant. A rectal examination reveals bloody stool and no masses. Which of the following is the most likely diagnosis?

  A. Arteriovenous malformation

  B. Diverticulitis

  C. Diverticulosis

  D. Ischemic colitis

  E. Ulcerative colitisExplanation:The correct answer is D. Thiselderly woman with a history of atherosclerotic vascular disease as demonstratedby a history of a myocardial infarction and exertional angina has developedcolitic symptoms, as demonstrated by the left lower quadrant pain and bloodydiarrhea. This is typically due to ischemia of small branches of the inferiormesenteric artery. The diagnosis is suspected clinically and generally confirmedwith a flexible sigmoidoscopy, since many cases involve the rectosigmoid region.Arteriovenous malformation (choice A) producespainless bleeding in the elderly, most commonly located in the cecum and often,but not always, associated with aortic stenosis.Diverticulitis (choice B) doespresent with postprandial crampy left lower quadrant abdominal pain and fever,but does not produce bloody diarrhea. Often, these patients have symptomsof constipation.Diverticulosis (choice C) alsopresents with postprandial crampy left lower quadrant pain or may be asymptomatic,but does not produce a bloody diarrheal illness.Ulcerative colitis (choice E) typicallypresents in a subacute or chronic fashion in a younger patient population.

Page 8: TEST 21

Although it can occur in this patient's age group, it rarely presents as anacute illness.7A 31-year-old woman complains of severe low back pain after playingtennis for 4 hours. She begins taking ibuprofen every 3 hours but finds littlerelief. She is prescribed codeine for her pain relief. Her physician alsoprescribes misoprostol, since the patient has a history of a bleeding gastriculcer 1 year earlier after taking ibuprofen. She has also been taking sumatriptanfor frequent migraine headaches. Which of the following is the most appropriate advice for thispatient?

  A. Avoid dietarypotassium

  B. Increasedietary roughage

  C. Use aneffective method of contraception

  D. Increaseher daily folate intake

  E. Increaseher daily aerobic exercise activityExplanation:The correct answer is C. Thispatient has been appropriately prescribed misoprostol for the prevention ofNSAID-induced ulcers, given the history that she has bled once previouslywith the use of NSAIDs. Since misoprostol is a prostaglandin analogue, itwill counter the effect of NSAIDs on the gastric mucosa. However, as a prostaglandinanalogue, it will cause smooth muscle contractions, including within the uterus,which can lead to a spontaneous abortion in a pregnant woman. She should thereforebe advised to maintain adequate birth control measures while using misoprostol.None of the other choices (choices A,B, D, or E) have particular relevance to this patient's health concerns.8A 20-year-old man is brought to the clinic by his parents, who are concernedthat he is an alcoholic. They want him to check himself in for treatment.In a discussion about his drinking habits, the young man brags that he can

Page 9: TEST 21

now drink more before he gets drunk. Which of the following terms best characterizes this phenomenon?

  A. Blackout

  B. Confabulation

  C. Dereism

  D. Resistance

  E. ToleranceExplanation:The correct answer is E. Tolerancedescribes the phenomenon of a drinker needing greater amounts of alcohol toget the same effect. It develops over time and is an indication of dependence.A blackout (choice A) occursduring a period of alcohol intoxication for which there is a complete anterogradeamnesia even though patient is awake and alert.Confabulation (choice B) isassociated with Korsakoff syndrome and is a result of chronic alcohol abuse.Because of retrograde and anterograde amnesia, the person fills in gaps inmemory by imagined or untrue experiences that he or she believes happened,even though they have no basis.Dereism (choice C) is mentalactivity that is not in accordance with reality or logic. It is mostly seen in psychoses.Resistance (choice D) is aphenomenon associated with the development of transference. It refers to inhibitionof free association and an opposition of goals of analysis.9A 29-year-old woman comes to the physician because of ulcers in hergenital area. She states that she first noticed the ulcers a few days ago.The ulcers are not painful, but she has had headache, fever, and loss of energyfor the past week or so. She has no other medical problems and has never hadsurgery. She has no known drug allergies. Examination reveals several small(~5 mm), shallow ulcers on the posterior vulva and enlarged and tender lymphnodes in the femoral, inguinal, and perirectal areas. Laboratory evaluationdemonstrates a negative rapid plasma reagin (RPR) test. Antibody testing revealselevated IgM and IgG titers to Chlamydia trachomatis. Which of the following is the most likely diagnosis?

Page 10: TEST 21

  A. Chancroid

  B. Condylomaacuminata

  C. Lymphogranulomavenereum

  D. Mononucleosis

  E. SyphilisExplanation:The correct answer is C. Lymphogranulomavenereum is a sexually transmitted disease caused by Chlamydiatrachomatis subtypes L1, L2, and L3. It is rare in the UnitedStates, but common in developing countries. Patients with lymphogranulomavenereum have ulcers that develop at the site of infection. These ulcers arenot painful and may go unnoticed by the woman. Together with the ulcers, thepatient may develop lymphadenitis that can involve the femoral, inguinal,perirectal, or pelvic lymph nodes, depending on the site of the initial infection.If the disease goes untreated, the lymph node involvement can lead to chronicinflammation, draining sinuses, strictures, and fistulae. Diagnosis can bemade through antibody testing or culture. Treatment is with doxycycline or erythromycin for 3 weeks. Chancroid (choice A) alsocan present with genital ulcers. The ulcers of chancroid are painful, however,and a patient with chancroid would not be expected to have elevated antibodytiters to Chlamydia trachomatis.Patients with condyloma acuminata (choiceB) (genital warts) complain of superficial growths that may bepruritic. These warts are caused by the human papillomavirus. Mononucleosis (choice D) isan infectious disease caused by the Epstein-Barr virus. It typically causesfever, sore throat, lymphadenopathy, and malaise. It is not associated withgenital ulcers.Syphilis (choice E) is associatedwith painless genital ulcers. Patients with syphilis, however, would be expectedto have a positive rapid plasma reagin test and would not be expected to haveelevated antibody levels to Chlamydia trachomatis.10A 54-year-old obese man gives a history of burning retrosternal pain

Page 11: TEST 21

and heartburn that is brought about by bending over, wearing a tight belt,or lying flat in bed at night. He gets symptomatic relief from over-the-counterantacids or H2 blockers, but has never been formally studied or treated. The problem has been present for many years and seems to be progressing. He doesnot smoke cigarettes and only drinks alcohol when he watches football gameswith friends, which only occurs a few weekends a year. Which of the following is the most appropriate next step in management?

  A. Barium swallow

  B. Cardiac enzymes and electrocardiogram

  C. Proton pump inhibitors

  D. Endoscopy and biopsies

  E. Laparoscopic Nissen fundoplicationExplanation:The correct answer is D. Theclinical picture is fairly convincing for long-standing gastroesophageal reflux.The main concern is the degree of peptic esophagitis that he may have developed,and the possibility of Barrett's esophagus and premalignant changes. Endoscopyand biopsies will provide the answer.Barium swallow (choice A) wouldprovide anatomic evidence of hiatal hernia and evidence of reflux, but wouldnot tell us whether Barrett's esophagus has developed.Cardiac enzymes and an electrocardiogram (choiceB) would be part of the work-up (along with pH monitoring) if wewere uncertain as to the genesis of ill-defined low retrosternal and upperepigastric pain. This man gives a classic presentation for reflux.Proton pump inhibitors (choice C) mightlikewise be indicated for this man, but not until we know the severity andpotential premalignant stage of his disease.Nissen fundoplication (choice E) maysome day be needed here, but one would not jump to a surgical solution basedonly on a clinical presentation.11A 44-year-old man comes to the clinic complaining of weakness in hisarms. On further discussion it seems that recently he also has begun to sufferfrom leg cramps and “muscle twitches.” Indeed, while speaking to you he displayssmall fasciculations on one side of his face. These symptoms began innocuously

Page 12: TEST 21

sometime during the preceding year and gradually became worse. Nothing seemsto help and the patient now is having difficulty swallowing and has occasionalslurred speech. He denies any bowel or bladder dysfunction, has no pain associatedwith his symptoms, and denies any abnormal sensations. Physical examinationreveals weakness of the proximal limb muscles and tongue, hyperactive reflexes,and spasticity. Sensory examination is normal. The patient is sent for cognitive and electrodiagnostic studies. Although no cognitive deficits are present,electromyogram results are found to be consistent with amyotrophic lateralsclerosis (ALS). The severity of the diagnosis is discussed, as are copingstrategies and appropriate support services. Which of the following is appropriate counseling about what thepatient can expect in the next few years?

  A. Bladder and bowel dysfunction is a common complication

  B. Dementia and mental status changes will become more pronounced

  C. His children each have a 50% chance of developing the disease

  D. Respiratory failure and death within 3 to 5 yearsis common

  E. Sensory deficits and neuropathic pain are late findingsExplanation:The correct answer is D. ALSis a progressive motor neuron disease that presents with a combination oflower and upper motor neuron findings. Initial presentation is often withmuscle weakness and fasciculations. The median survival after diagnosis is3 to 5 years, with death usually occurring from respiratory failure.Although ALS presents with a combination of upper and lower motor neuronsigns, the disease does not occur with bowel or bladder involvement (choice A), prominent mental status changes (choice B), or sensory symptoms (choice E). The disease is limited primarilyto corticospinal and motor nerve pathology.Only 5 to 10% of cases are familial ALS. Although these cases are usuallyautosomal dominant, at this point nothing can be said about this patient’schildren. Their chance of developing the disease (choiceC) is unknown.12A 37-year-old woman comes to the physician because of severe heartburnwith or without meals. She has a history of hypertension, which has been treated

Page 13: TEST 21

with captopril. She also has a history of Raynaud disease, multiple facialtelangiectasias, and very taut skin on the dorsum of both hands. She has failedto obtain relief for her heartburn with large doses of antacids, ranitidine,or omeprazole. Esophageal manometry is ordered. Which of the following would be the most likely results of thistest?

  A. Decreased esophageal peristalsis and decreasedLES pressure

  B. Decreased esophageal peristalsis and increasedLES pressure

  C. Increased esophageal peristalsis and decreasedLES pressure

  D. Increased esophageal peristalsis and increasedLES pressure

  E. Normal esophageal peristalsis and normalLES pressureExplanation:The correct answer is A. Thispatient has the classic presentation of gastroesophageal reflux disease (GERD)in association with scleroderma. These patients have the deposition of collagenin the body of the esophagus, as well as the lower esophageal sphincter (LES).This results in the typical pattern of decreased esophageal peristalsis andthe reduced ability of the LES to maintain its high pressures between swallowing.These patients are therefore at risk for severe GERD and subsequent complicationsof peptic stricture and Barrett's esophagus. Although there is no corrective therapy to improve esophageal motility or increase LES pressure, aggressivetreatment is generally aimed at reducing acid production with the use of highdoses of proton pump inhibitors. Nevertheless, many of these patients developthe long-term consequences of GERD.Choice B suggests achalasia,in which impaired esophageal peristalsis is often accompanied by a lack oflower esophageal sphincter relaxation.Choice C doesn't describeany of the more common esophageal motor disorders.Choice D suggests symptomaticdiffuse esophageal spasm, particularly if the peristaltic waves were poorly

Page 14: TEST 21

organized.Choice E would be seen inpatients without esophageal motor disease.13A 32-year-old man comes to the physician complaining of severe pruritusover the past 2 weeks. He has a history of ulcerative colitis for the past7 years, which has remained well controlled on sulfasalazine and cortisoneenemas. His physical examination is unremarkable except for evidence of diffuseexcoriations on his extremities and trunk. Laboratory studies reveal a mildiron deficiency anemia and normal electrolytes. Liver function tests are normal,except for an alkaline phosphatase that is 322 U/L (normal, <110 U/L). Which of the following is the most likely explanation for his symptoms?

  A. Erythema nodosum

  B. Hepatitis C

  C. Primary biliary cirrhosis

  D. Primary sclerosing cholangitis

  E. Pyoderma gangrenosumExplanation:The correct answer is D. Thispatient has had longstanding ulcerative colitis and has now developed pruritusin the setting of an elevated alkaline phosphatase. This is consistent witha diagnosis of primary sclerosing cholangitis, whose activity is not relatedto the activity of the associated ulcerative colitis. This sclerosing processinvolves both the intra- and extrahepatic ducts and is diagnosed by endoscopicretrograde cholangiopancreatography (ERCP). Primary sclerosing cholangitisoccurs most often in young men and is commonly associated with inflammatorybowel disease, particularly ulcerative colitis. Classically, primary sclerosingcholangitis produces a triad of progressive fatigue, pruritus, and jaundice,although some patients will present with upper quadrant pain, fever, hepatosplenomegaly,or cirrhosis. The condition is worrisome because it may eventually progressto decompensated cirrhosis, portal hypertension, ascites, and liver failure.Treatment is generally supportive, with more specific measures as needed includingantibacterial treatment for superimposed bacterial cholangitis, dilation byendoscopy or a transhepatic route of significant strictures, and ursodeoxycholicacid to relieve the pruritus. A variety of anti-inflammatory therapies (e.g.,corticosteroids, azathioprine, methotrexate) have been tried but appear to

Page 15: TEST 21

have more adverse than beneficial effects. Liver transplantation appears tobe the only true cure.Although erythema nodosum and pyoderma gangrenosum can be skin conditionsseen in association with ulcerative colitis, they do not present with pruritusand, furthermore, have characteristic findings on physical examination. Erythemanodosum (choice A) presents as tender, red nodules, typically found on the lower extremities. Pyoderma gangrenosum (choice E) are pustular, ulcerating lesions,also generally found on the extremities, which can be very painful.There is no evidence of hepatocellular dysfunction or transaminase elevation,nor any history of hepatitis risk factors, so hepatitis C is unlikely (choice B).Primary biliary cirrhosis (choice C) doesin fact present with pruritus and an elevated alkaline phosphatase; however,it is typically seen in middle-aged women and has no association with ulcerativecolitis.14A 32-year-old man is brought into the emergency department by a friendbecause of a heroin overdose. This is the 3rd time that he is brought to thishospital for this reason and the friend reports that the patient has beenthrough several drug rehabilitation programs. He relapses each time he losesa job, which is quite frequently. After the administration of naloxone and a complete physical examination,which of the following laboratory studies is most important in the evaluationof this patient to provide long-term follow-up care?

  A. Albumin level

  B. Echocardiogram

  C. Electroencephalogram

  D. HIV antibody test

  E. Plasma liver enzyme levelsExplanation:The correct answer is D. Becauseof the frequent use of needles in a patient with heroin dependence, it isabsolutely necessary to rule out the possibility of HIV infection. This isespecially true with a patient who is unable to provide a history during anepisode of heroin overdose. Concomitant HIV infection and the need for diagnosiswill make this laboratory study the most important of all the studies listed.Albumin level (choice A) and

Page 16: TEST 21

other plasma liver enzyme levels (choice E) arenot usually affected by an episode of heroin intoxication.An echocardiogram (choice B) maybe indicated in a chronic IV drug abuser if there is evidence of a murmuron auscultation of the heart, pointing to the possibility of bacterial endocarditis.An electroencephalogram (choice C) isnot necessarily indicated in a patient with heroin intoxication.15A 38-year-old woman, gravida 3, para 2, at 32 weeks' gestation comesto the physician because of bleeding from the vagina. She states that thismorning she passed 2 quarter-sized clots of blood from her vagina. Otherwise,she states that she is feeling well. The baby has been moving normally andshe has had no contractions or gush of fluid from the vagina. Her obstetricalhistory is significant for 2 low-transverse cesarean deliveries for non-reassuringfetal heart rate tracings. An ultrasound is performed that demonstrates acomplete placenta previa. For which of the following conditions is this patient at highestrisk?

  A. Dystocia

  B. Intrauterinefetal demise (IUFD)

  C. Placentaaccreta

  D. Preeclampsia

  E. UterineruptureExplanation:The correct answer is C. Placentaprevia is defined as a placenta located over the cervical os. There are 3major types. Complete previa describes a placenta that completely covers thecervical os. Partial previa is a placenta that covers some of the cervicalos, with the remainder of the os uncovered by the placenta. Marginal previadescribes a placenta that is located at the edge of the cervical os. Threemajor risk factors for placenta previa are maternal age, minority race, and previous cesarean delivery. Placenta accreta describes the condition in whichthere is abnormal attachment of the placenta to the uterine wall. In thiscondition, the decidua basalis is absent and the placenta is attached to the

Page 17: TEST 21

myometrium (accreta) or invades into the myometrium (increta), or perforatesthrough the myometrium (percreta). Patients with a placenta previa and noprior cesarean deliveries have a 5% risk of having a placenta accreta. Patientswith a previa and one prior cesarean delivery have a 25% risk of having anaccreta. And patients with a placenta previa and 2 or more prior cesareandeliveries have a greater than 50% risk of having a placenta accreta. Manypatients with a previa and accreta will require a hysterectomy at the timeof delivery. This patient, with a history of 2 prior cesarean deliveries anda placenta previa is at highest risk for placenta accreta.This patient would not be considered to be at highest risk of dystocia (choice A) because, with a placenta previa,she would not be allowed to labor and, therefore, would not be at risk ofdystocia.There appears to be no strong association between placenta previa andintrauterine fetal demise (choice B)or preeclampsia (choice D).Uterine rupture (choice E)is a concern in women who have had prior cesarean deliveries, and the riskof rupture does rise with the number of previous cesarean deliveries. It isof particular concern if the woman is in labor. This patient, with a placentaprevia, however, will not be allowed to labor.16A young man sustains a gunshot wound to the base of his neck. He wasshot point blank with a .38 caliber revolver. The entrance wound is abovethe left clavicle, below the level of the cricoid cartilage, and just lateralto the sternomastoid muscle. The exit wound is just above the spinous processof the right scapula. He has normal breath sounds on both sides, is awakeand alert, is talking with a normal tone of voice, is neurologically intact,and is hemodynamically stable. Portable x-ray films of the neck and chesttaken in the emergency department show some air in the tissues of the lowerneck, but are otherwise non-diagnostic. Which of the following is the most appropriate next step in management?

  A. Observationfor several hours

  B. CT scanof the lower neck and upper chest

  C. Angiogram,esophagogram, esophagoscopy, and bronchoscopy prior to surgical exploration

 

Page 18: TEST 21

D. Immediatesurgical exploration of the lower neck through a collar incision

  E. Immediatesurgical exploration of the upper chest through a median sternotomyExplanation:The correct answer is C. Gunshotwounds to the base of the neck need exploratory surgery, but the exact approachand incision are determined by a more accurate knowledge of the location and extent of the injuries. Thus, if time permits, diagnostic studies should precedesurgical intervention. The major vessels, the tracheobronchial tree, and theesophagus are the potential targets that have to be investigated.Observation (choice A) mightbe appropriate for a stab wound in a completely asymptomatic patient. In gunshotwounds, we have to expect that injuries will exist, and they should not beneglected waiting for overt clinical signs.CT scan (choice B) has donewonders for our assessment of closed head injuries and blunt abdominal trauma,but it is not the study that would tell us what has happened to the majorvessels, the esophagus, or the tracheobronchial tree in a gunshot wound.Immediate surgical exploration, either through the neck or the chest,or in combination, might be forced by a rapidly deteriorating situation. Inthe absence of such imperative, a decision to open the neck (choice D) or the chest (choiceE) is premature at this point.17A 23-year-old medical student returns from summer vacation in the Caribbean,where he worked as a volunteer in a local emergency department. During hislast week in Jamaica, he developed watery diarrhea and mild abdominal cramping.The symptoms have persisted since his return to the U.S. At present, he reportsthat he is having up to 10 watery, non-bloody bowel movements per day and poorly localized abdominal discomfort. On physical examination, he is afebrileand there is no abdominal tenderness. His stool is light brown and guaiacnegative. Which of the following is the most likely explanation for his symptoms?

  A. Enteroinvasive Escherichiacoli

  B. Enterotoxigenic E. coli

  C. E. coli 0157:H7

Page 19: TEST 21

  D. Tropical sprue

  E. Vibrio choleraeExplanation:The correct answer is B. Thispatient presents with the typical findings of a secretory diarrhea causedby an ingested toxin. In this case, enterotoxigenic Escherichiacoli is the most common cause of traveler's diarrhea. Itrarely requires antibiotics, and patients will generally respond to conservativemanagement.Enteroinvasive E. coli (choiceA) produces a bloody diarrhea with fevers and findings of abdominal tenderness. In these patients, the E. coli causesmucosal ulceration, resulting in the bloody diarrhea and mucopurulent stoolsoften seen.E. coli 0157: H7 (choiceC) presents with the appearance of severe toxicity and symptomsof hemolysis and uremia, i.e., the hemolytic uremia syndrome. It is usuallyfound as part of an epidemic after ingestion of contaminated beef.Tropical sprue (choice D) mayoccur in patients returning from the Caribbean but will generally presentwith signs and symptoms of malabsorption, particularly B12, which are notdescribed here.Vibrio cholerae (choiceE) can produce a clinical picture identical to infection with enterotoxigenic E.coli, but it does not occur in the Caribbean. Instead, itis far more common on the Indian and African continents. Rare cases are foundalong the Gulf Coast of the U.S.18A 37-year-old accountant comes to the physician to ask for advice regardingthe future management of his ulcerative colitis. He has had pancolitis forthe past 19 years and has been told that he is at an increased risk for developingcolorectal cancer. He asks for the physician's recommendation regarding appropriatesurveillance. Which of the following is the most appropriate response?

  A. Annual stool guaiac testing

  B. Barium enema

  C. Colonoscopy

Page 20: TEST 21

  D. Colonoscopy and multiple biopsies

  E. Flexible sigmoidoscopy with multiple biopsiesExplanation:The correct answer is D. Patientswith longstanding extensive ulcerative colitis for at least 10 years' durationare at increased colon cancer risk. Appropriate surveillance involves annualor biannual colonoscopy with multiple biopsies at regular intervals, evenof normal appearing mucosa, to check for dysplasia.None of the other choices allows sampling of the entire colonic mucosafor histologic examination for the precancerous lesion of low- or high-gradedysplasia.19A patient is talking to his psychiatrist about a conflict he has withhis partner. They argue because the partner complains that the patient isinefficient and procrastinates doing things that are his responsibility. Henever completes a task the way in which he was asked to do it, but in hisown way and usually much later. Which of the following types of defense mechanism is this patientexhibiting?

  A. Acting out

  B. Blocking

  C. Passive-aggressiveness

  D. Regression

  E. SplittingExplanation:The correct answer is C. Withpassive-aggressiveness, aggression toward others is expressed indirectly,usually through procrastination, stubbornness, passivity, and forgetfulness.The manifestations usually affect others more than the person who is doingit.Acting out (choice A) is adefense by which a person expresses an unconscious wish through action to

Page 21: TEST 21

avoid being conscious of the strong accompanying affect.Blocking (choice B) is a transientinhibition of thinking that usually involves strong affects and impulses thatare being inhibited.Regression (choice D) is adefense mechanism in which a person returns to an earlier stage of libidinaldevelopment to avoid tension at the present level of development.Splitting (choice E) is adefense mechanism by which external objects are divided into "all good" and"all bad." It is accompanied by abrupt shifting of an object from one categoryto the other.20A 62-year-old woman comes to the physician because of bleeding fromthe vagina. She states that her last menstrual period came 11 years ago andthat she has had no bleeding since that time. She has hypertension and type2 diabetes mellitus. Examination shows a mildly obese woman in no apparentdistress. Pelvic examination is unremarkable. An endometrial biopsy is performedthat shows grade I endometrial adenocarcinoma. Which of the following is the most appropriate next step in management?

  A. Chemotherapy

  B. Cone biopsy

  C. Dilationand curettage

  D. Hysteroscopy

  E. HysterectomyExplanation:The correct answer is E. Endometrialcancer is the most common gynecologic cancer in women ages 45 and older. Themain factor that predisposes a woman to the development of endometrial canceris exposure to unopposed estrogen, whether endogenous or exogenous. Endogenousfactors include, early menarche, late menopause, chronic anovulation, estrogen-secretingovarian tumors, and obesity. Exogenous factors include the ingestion of unopposedestrogen (as with estrogen replacement therapy). Hypertension and diabeteshave also been associated with endometrial cancer, though this relationshipmay likely be related to obesity. This patient has endometrial cancer on thebasis of her endometrial biopsy result. The correct management for this patient

Page 22: TEST 21

is with total abdominal hysterectomy, bilateral adnexectomy, and possiblelymph node sampling.Chemotherapy (choice A) wouldnot be the most appropriate next step in management. If the patient were nota surgical candidate, because of her obesity, for example, then radiationtherapy could be administered.Cone biopsy (choice B) isused in the diagnosis and management of cervical cancer. It would not be usedfor this patient with an endometrial biopsy showing endometrial cancer.Dilation and curettage (choice C)or hysteroscopy (choice D) wouldnot be the most appropriate next step in management. The diagnosis of endometrialcancer has been made on the basis of the endometrial biopsy. Therefore, themost appropriate next step in management is to treat the patient through hysterectomyor, if hysterectomy is not possible because of obesity or medical disease, radiation.21A 58-year-old man undergoes a liver transplantation. The procedure goeswell, without any noted complications, however, 10 days later the patient'slevels of gamma-glutamyl transferase (GGT), alkaline phosphatase, and bilirubinbegin to rise. Which of the following is the most appropriate next step in diagnosis?

  A. Measurement of preformed antibody levels

  B. Ultrasound of biliary tract and Dopplerstudies of the anastomosed vessels

  C. Liver biopsy and determination of portalpressures

  D. Liver biopsy and more detailed liver functiontests

  E. Liver biopsy and trial of steroid bolusesExplanation:The correct answer is B. Inall other solid organ transplants, deterioration of function 10 days out wouldsuggest an acute rejection episode, and appropriate biopsies would be doneto confirm the diagnosis. In the case of the liver, however, antigenic reactionsare less common, whereas technical problems with the biliary and vascularanastomosis are the most common cause of early functional deterioration. Theyare, therefore, the first anomalies to be sought.

Page 23: TEST 21

Preformed antibodies (choice A) areresponsible for hyperacute rejection, which would be evident within minutesof establishing blood flow to the graft.Choices C, D, and E are centered on liver biopsy, which wouldbe done only after technical problems have been ruled out.22A 54-year-old woman complains of severe lower abdominal pain and distention.The symptoms began approximately 24 hours ago, when her abdomen became visiblyswollen and she developed nausea and vomiting. She has not moved her bowelsover the past 24 hours. Over the past 4 months, she has lost 14 pounds andhas noted progressive symptoms of constipation. On several occasions, shehas noted blood mixed in with her bowel movements, which have become thinnerin caliber. She denies any recent travel, use of antibiotics, or fevers. Onphysical examination, she appears acutely uncomfortable and has a temperatureof 38.3 C (100.9 F). Her abdomen is diffusely distended and tender to palpationin the left lower quadrant. There are hyperactive rushing bowel sounds. Onrectal examination, her stool is brown and guaiac positive. An obstructiveseries reveals multiple small bowel air fluid levels and a dilated colon proximalto the sigmoid colon. Which of the following is the most likely diagnosis?

  A. Amebic abscess

  B. Colon polyp

  C. Diverticulitis

  D. Diverticulosis

  E. SigmoidcarcinomaExplanation:The correct answer is E. Thispatient has symptoms of a chronic gastrointestinal process as demonstratedby her weight loss, change in bowel habits, and thinner caliber stools withbleeding. The thinner caliber stools specifically suggest that a mass lesionor luminal narrowing is present. Of the lesions listed, only colonic cancerwould be likely to produce this pattern. In other settings, Crohn diseaseand tuberculosis of the colon could produce similar clinical patterns. Thispatient's change in symptoms over the past 24 hours suggests that she mayhave developed an acute large bowel obstruction.An amebic abscess (choice A)

Page 24: TEST 21

would be expected to give more focal findings and symptoms of high spikingfevers. Furthermore, there would usually be a history of travel to suggestthis diagnosis.A colon polyp (choice B) willvery rarely produce obstructive symptoms and does not produce the symptomsof weight loss and persistent thin caliber stool.Diverticulitis (choice C)is a result of a micro- or macroperforation of a diverticulum and resultswith an acute onset of local inflammatory signs, usually in the left lowerquadrant, corresponding to the sigmoid colon diverticulosis.Diverticulosis (choice D)may give her crampy lower abdominal pain and, on rare occasion, may produceobstructive symptoms. However, this patient's symptoms of weight loss andbloody bowel movements with thinner caliber stools are more suggestive ofcolon carcinoma.23A 34-year-old woman complains of difficulty swallowing both liquidsand solids for the past 6 months. She has a history of hypertension and Raynaudphenomenon. She drinks 1 glass of wine with dinner every night and does notsmoke cigarettes. A physical examination reveals tight skin on her face and on the dorsal surfaces of both hands. Which of the following manometric findings would most likely be found in this patient?

Peristalsis in body of esophagus pressureResting lower esophageal sphincter (LES)

  A.DecreasedNormal

  B.DecreasedIncreased

  C.IncreasedDecreased

  D.DecreasedDecreased

Page 25: TEST 21

  E.IncreasedIncreasedExplanation:The correct answer is D. Scleroderma(progressive systemic sclerosis) is an important disease in which a varietyof body tissues can undergo fibrosis. The most obvious of these is the skin,which produces the tight thick skin seen on this patient's hands and face.Internally, the esophagus is particularly susceptible to this fibrotic process,which will destroy the ability of the esophagus to undergo peristalsis, becauseof both the loss of muscle and the stiffening by fibrosis. Typical manometricfindings in these patients are diminished or absent peristalsis in the bodyof the esophagus with a greatly reduced resting lower esophageal sphincter(LES) pressure. Scleroderma patients are at significant risk for severe gastroesophagealreflux disease (GERD) and its complications.Choices A and C are not typicalof any condition you need to remember.Associate the findings illustrated in ChoiceB with achalasia.Associate the findings illustrated in ChoiceE with symptomatic diffuse esophageal spasm.24A 32-year-old man is in twice-weekly insight-oriented psychotherapywith a psychiatrist. Recently, the patient has been exploring his thoughtsand feelings around his wife's complaint that he is too restricted and inhibitedin their sexual activity. The patient admits that he wishes to be more sexually available for his wife, but finds himself maintaining a restricted stance. Which of the following defense mechanisms would best describe thispatient's tendency in his sexual relationship with his wife?

  A. Projection

  B. Reactionformation

  C. Sexualization

  D. Somatization

 

Page 26: TEST 21

E. SublimationExplanation:The correct answer is B. Reaction formation, often seenin obsessional characters, is the term for the defense mechanism in whichan unacceptable impulse is transformed into its opposite. In this case, duringinsight-oriented psychotherapy, the patient realizes his wish to be freerin his sexual relationship with his wife (an impulse which he finds unacceptableon some level) but finds himself responding in the opposite way (maintaininga restricted stance). Inhibition may also partly account for this man's difficulty,in that a renunciation is used to evade anxiety arising out of impulses. Projection (choice A) occurswhen an unacceptable inner impulse is perceived and reacted to as though itwas outside oneself. On the psychotic level, this takes the form of delusionsand hallucinations. Sexualization (choice C) occurswhen an object or function is endowed with sexual significance that it didnot previously have in order to ward off anxieties associated with prohibitedimpulses. Somatization (choice D) describesthe defense mechanism that occurs when emotional concerns are converted intobodily symptoms, and the person tends to react with somatic manifestations. If the patient in this case had a tendency to use somatization, he mightunconsciously use physical symptoms to get rid of the anxiety around his conflictedsexual thoughts. Sublimation (choice E) isa mature defense mechanism that occurs when a socially acceptable means ofexpressing an impulse replaces one that would be socially unacceptable. Sublimation allows instincts to be channeled, rather than blocked or diverted. Feelingsare acknowledged, modified, and directed toward a significant object or goal,and modest instinctual satisfaction occurs.25A 26-year-old pregnant woman comes to the physician complaining of palpitationsand anxiety. She is currently at 4 months' gestation and has had no complicationsthus far. She denies any other complaints. Her temperature is 37.0 C (98.6F), blood pressure is 110/80 mm Hg, pulse is 83/min, and respirations are18/min. Physical examination is normal. There is no tremor, exophthalmus,or myxedema. Her thyroid gland is palpable but not enlarged. Laboratory studiesshow:

Which of the following is the most appropriate intervention atthis time?

  A. Methimazole

Page 27: TEST 21

  B. Propylthiouracil

  C. Radioactiveiodine

  D. Reassurance

  E. SubtotalthyroidectomyExplanation:The correct answer is D. Thispatient is euthyroid. This is easily recognized by her normal thyroid-stimulatinghormone value. If a thyroid-stimulating hormone value is normal, no otherthyroid function tests are necessary because it is the most sensitive testof thyroid function. This patient's palpitations are not likely related toher thyroid gland. Our patient's total T4 is elevated because the increasedestrogen concentration in her body that occurs during pregnancy leads to anincrease in thyroid-binding globulin. Oral contraceptives also increase thyroid-bindingglobulin. Patients will have increased total T3 and T4 but free hormone andthyroid-stimulating hormone values are normal. Methimazole (choice A) andpropylthiouracil (choice B) areantithyroid drugs that work by decreasing iodination of thyroid hormone inthe thyroid gland and therefore decrease levels of functional thyroid hormone.Propylthiouracil also decreases peripheral conversion of T4 to its active form, T3. These drugs are not appropriate because our patient is euthyroid.In addition, methimazole is not safe to use during pregnancy. Propylthiouracilis safe during pregnancy. Radioactive iodine ablation (choice C) of the thyroid gland is the treatment of choice in most patientswith hyperthyroidism. It is well tolerated and effective. It cannot be usedin pregnant patients because it will affect the fetus's thyroid gland, leadingto birth defects and developmental problems.Subtotal thyroidectomy (choice E)is a treatment option only for hyperthyroid patients who are in their secondtrimester of pregnancy or hyperthyroid children. Again, our patient is euthyroidand does not need any intervention at this time.26A 67-year-old obese man is diagnosed with type 2 diabetes mellitus ata screening conducted at a local shopping center and is then referred to aninternist. In the comprehensive evaluation that ensues, several abnormalities

Page 28: TEST 21

are uncovered in addition to the diabetes: He has an alkaline phosphatase4 times the upper limit of normal, his total bilirubin is 2.7 mg/dL, he hasa hemoglobin of 9 mg/dL, and his stools are positive for occult blood. A sonogramof his upper abdomen shows dilated intrahepatic and extrahepatic ducts. Which of the following is the most appropriate next diagnostictest?

  A. CT scanof the abdomen

  B. Colonoscopy

  C. Endoscopicexamination of the duodenum

  D. Percutaneoustranshepatic cholangiogram

  E. Upper gastrointestinalseries with bariumExplanation:The correct answer is C. Theman obviously has early obstruction of his biliary tract, of which he hadnot been clinically aware. He also has blood in the lumen of the gut, andhas been bleeding sufficiently to become anemic. The only single lesion thatcould account for both if these findings is a cancer of the ampulla of Vater.Endoscopic examination should reveal it and allow biopsies to confirm thediagnosis. CT scan of the abdomen (choice A)can reveal sources of biliary obstruction if they are large (cancer of thehead of the pancreas) or calcified (gallstones), but is unlikely to show asmall ampullary cancer.Colonoscopy (choice B) wouldhave been the thing to do if all he had was anemia and blood in the lumenof the gut, in which case the diagnostic suspicion would have been cancerof the right colon. But cancer of the colon could not account for the biliaryobstruction. Percutaneous cholangiogram (choice D) is sometimes done to visualize the biliary tract, but ERCP isusually preferred. In any event, we should be able to establish the diagnosisin this case by just doing the "E" of the ERCP (endoscopy).

Page 29: TEST 21

Upper gastrointestinal series (choiceE) is a poor second choice to direct visualization when trying to confirm the presence of an ampullary cancer.27A 45-year-old woman with bipolar disorder type I comes to the physicianbecause of worsening of her depressive symptoms and weight gain. She had beenstable for a while but lately has started to feel tired and slow. Her skinhas been drier than usual and her husband noticed a change in her voice. Sheis concerned that she cannot tolerate stress any more, because her hair isfalling out. After talking to her and reviewing her medications, the physicianruns some tests. After the results are back, the physician's suspicion isconfirmed. Based on the history, this patient is most likely being treatedwith which of the following medications?

  A. Clonazepam

  B. Gabapentin

  C. Lithium

  D. Olanzapine

  E. ValproicacidExplanation:The correct answer is C. Thispatient seems to have developed hypothyroidism, which is a side effect oflithium therapy. Symptoms of lithium toxicity consist of diarrhea, severetremor, polyuria, ataxia, confusion, and seizures, but less serious adverseeffects include gastric distress, weight gain, tremor, fatigue, and cognitivedeficits. Long-term treatment can result in hypothyroidism. Other side effectsinvolve kidneys, heart, and skin. Clonazepam (choice A) is abenzodiazepine that is used only in place of antipsychotics or in conjunctionwith 1 to treat acute manic agitation. It is used for short-term treatmentof anxiety, insomnia, or catatonia associated with mania or depression. Itcan also be used as adjunctive maintenance treatment together with mood stabilizers,but lacks specific antimanic, antidepressant, or mood stabilizing properties.It is not associated with hypothyroidism. Gabapentin (choice B) is anew antiseizure medication that is approved as adjunctive therapy of treatment-refractory

Page 30: TEST 21

partial seizures. In some studies and anecdotal reports it has been used as adjunctive medication for treatment of refractory bipolar disorder. Its moodstabilizing properties have not been confirmed. Most common side effects includesomnolence, dizziness, and ataxia. It is not associated with hypothyroidism.

Olanzapine (choice D) belongsto the group of atypical antipsychotics that also have some mood stabilizingproperties and lately have been used in treatment of bipolar disorders. Itsmost common side effects include postural hypotension, constipation, weightgain, and dizziness. Hair loss is not one of the side effects of this medication,however. Valproic acid (choice E) isan anticonvulsant medication that also has been approved by the Federal DrugAdministration for treatment of manic episodes. Its common side effects include nausea, sedation, weight gain, transitory hair loss, and transitory increasein liver function tests. Rare side effects are related to hepatitis, pancreatitis,and possible decrease in platelets or platelet dysfunction. This patient'ssymptoms are more consistent with hypothyroidism, which is associated withlithium.28A 23-year-old woman comes to the physician because of a swelling inher vagina. She states that the swelling started about 3 days ago and hasbeen growing larger since. The swelling is not painful, but it is uncomfortablewhen she jogs. She has asthma for which she uses an albuterol inhaler, butno other medical problems. Examination shows a cystic mass 4 cm in diameternear the hymen by the patient's left labia minora. The mass is nontender andthere is no associated erythema. The mass is freely mobile. The rest of thepelvic examination is unremarkable. Which of the following is the most likely diagnosis?

  A. Bartholin's cyst

  B. Condyloma lata

  C. Granuloma inguinale

  D. Hematocolpos

  E. Vulvar cancerExplanation:The correct answer is A. Thispatient has a presentation and findings that are most consistent with a Bartholin's

Page 31: TEST 21

cyst. Bartholin's cysts develop when a Bartholin's gland becomes obstructed.The Bartholin's glands are bilateral structures that are present near theposterior fourchette of the vagina at the 5 and 7 o'clock positions. Theysecrete mucus, particularly during sexual stimulation, which drains into theposterior vagina.They undergo rapid growth during the process of puberty andthey shrink after the menopause. When the duct of the Bartholin's gland becomesobstructed, a Bartholin's cyst results. If the cyst becomes infected, theresult is a Bartholin's abscess. These abscesses are usually polymicrobialin nature, although the gonococcus is implicated in about 25% of cases. Treatmentof a symptomatic Bartholin's cyst is with placement of a Word catheter. Thisis a small balloon-tipped catheter device that is placed into a small holethat is punched into the cyst itself. This catheter allows drainage of thecyst and the formation of an epithelialized tract that will allow continueddrainage once the catheter is removed. This tract should prevent the cystfrom reforming. If Bartholin's cysts continue to form in spite of the useof the Word catheter, a marsupialization procedure may be tried. In this procedure,the cyst walls are sutured open to the surrounding skin to prevent re-closureand re-formation of the cyst.Condyloma lata (choice B) isa manifestation of secondary syphilis. They appear as coalesced, large, pale,flat-topped papules and not as a cystic mass.Granuloma inguinale (choice C) isalso known as Donovanosis and is a sexually transmitted disease associatedwith the gram-negative bacillus Calymmatobacterium granulomatis.The disease is characterized by papules progressing to ulcers and not by avulvar cyst.Hematocolpos (choice D) describesthe condition in which there is blood filling the vagina. This is often seenwith an imperforate hymen.Vulvar cancer (choice E) doesnot usually present as a single cystic mass at the introitus and, in youngwomen, is far less common than Bartholin's cysts29A 33-year-old woman develops bloody nipple discharge from the rightbreast. It occurs intermittently over the course of 6 weeks, so she seeksmedical advice. There is no family history of breast cancer. The patient deniesany trauma, recent pregnancy, prior similar episodes, pain, or fevers. On examination, both breasts are normal in appearance. There are no palpablemasses in either breast. A very small amount of blood can be expressed fromthe right nipple. Which of the following is the most likely diagnosis?

  A. Breast abscess

  B. Cystosarcoma

Page 32: TEST 21

phyllodes

  C. Fibroadenoma

  D. Intraductalpapilloma

  E. MammarydysplasiaExplanation:The correct answer is D. Intraductalpapilloma is seen in women between the ages of 20 and 40 years who presentwith bloody nipple discharge. There are no palpable findings on physical examination.Other possible lesions must be ruled out, and this is done with mammogram.On mammogram, the papilloma will not show because of its small size. Resectionmay be performed for symptomatic relief. Galactogram may be helpful in guidingresection.A breast abscess is seen in lactating women only (choice A). It is associated with cracking of the nipple and ared, warm, fluctuant, tender mass. Treatment is incision and drainage. Ifthe woman is not lactating, carcinoma of the breast is a serious concern.Cystosarcoma phyllodes (choice B)are seen in women in their late twenties. These lesions grow over many yearsand become very large, which leads to distortion of the breast. They do notbecome fixed or invading, however. Most are benign but have the potentialfor malignant sarcomas.Fibroadenoma (choice C) isseen in women in their late teens and early twenties. There is a firm, rubberymass that moves easily with palpation. Diagnosis is made with fine needleaspirate or sonogram.Mammary dysplasia (choice E),also called fibrocystic disease or cystic mastitis, is seen in women in theirthirties and forties who have bilateral tenderness related to menstrual cycleand multiple lumps that seem to come and go.30The mother of a 35-year-old man calls her primary care physician foradvice on a drug treatment program for her son. She states that he lives withher since the loss of his job and apartment four months ago. She reports thathe goes out on weekends and uses drugs heavily. When he returns home, usuallyon Monday mornings, he sleeps for the entire day and seems very depressed.He is also very irritable and anxious over the course of the next day. Sheknows of no other withdrawal symptoms. She believes that he uses the drugthroughout the week, but with more intense binges on the weekends. On several

Page 33: TEST 21

occasions, he has called her from the local psychiatric emergency center afteradmitting himself for suicidal ideation after use of the drug. Which of the following drugs is most likely responsible for thisman's withdrawal state?

  A. Alcohol

  B. Benzodiazepines

  C. Cannabis

  D. Cocaine

  E. HeroinExplanation:The correct answer is D. Theuse of cocaine, especially crack cocaine (an extremely potent, freebase formthat is sold in small, ready-to-smoke amounts) is associated with the withdrawal symptoms described in this case. After cessation of the use of cocaine, orafter acute intoxication, a post-intoxication depression (crash) often occurs.This depression is associated with dysphoria, anhedonia, anxiety, irritability,fatigue, hypersomnolence, intense craving, and suicidal ideation.Alcohol (choice A) is associatedwith a potentially dangerous set of withdrawal phenomena. These range frommild withdrawal (mild tremulousness) to a full withdrawal syndrome (tremulousness, diaphoresis, hyperreflexia, elevated vital signs, and anxiety) to the verydangerous withdrawal state known as delirium tremens (altered mental status,hallucinations, autonomic instability, seizures, and possibly death). Alcoholwithdrawal should be treated in a medical setting with close observation and a detoxification regimen.Benzodiazepines (choice B)can cause dependence and a wide range of withdrawal phenomena very similarto those of alcohol withdrawal. Benzodiazepine withdrawal should also be managedwith medical supervision.Cannabis (choice C) may beassociated with a withdrawal state including irritability, insomnia, increasedor decreased appetite, restlessness, tremor, increased reflexes, and changesin vital signs. The withdrawal phenomena of cannabis are less well definedthan many other drugs, and do not lead to dangerous medical sequelae. Heroin (choice E) dependenceleads to a characteristic withdrawal syndrome that is very uncomfortable.

Page 34: TEST 21

Symptoms include myalgias, nausea, vomiting, diarrhea, lacrimation, rhinorrhea, piloerection, anxiety, insomnia, irritability, and intense craving.31A 38-year-old woman comes to the physician for an annual examinationand Pap smear. She has no complaints. She has a regular period every month.She is sexually active with her husband. She has migraine headaches and isstatus post a tubal ligation. She states that she uses numerous alternative medications for mood, sleep, and disease prevention. Examination, includingpelvic and breast examination, is unremarkable. Which of the following is an appropriate question to ask this patient?

  A. Does yourhusband know you are using these alternative medications?

  B. Do you realizehow dangerous alternative medicines are?

  C. Which alternativemedications do you use?

  D. Why don'tyou stick with traditional medicines?

  E. Why haven'tyou revealed your use of alternative medications before?Explanation:The correct answer is C. Someestimates indicate that roughly 50% of Americans use some forms of complementaryand alternative medicine (CAM). The categories of these include mind-body interventions, such as yoga, alternative systems of medical practice suchas Chinese medicine, pharmacologic treatments such as medicinal plants, herbalmedicine such as St. John's wort, diet therapies such as vegetarianism, manualhealing methods such as massage, and bioelectromagnetic applications suchas magnets for musculoskeletal pain. It is essential for the physician towork with the patient regarding the use of CAM. The first step is to findout which methods the patient uses. This patient has told the physician thatshe uses alternative medications. Many patients do not offer this information,assuming that the usual physician will not support CAM. It is therefore importantto ask the patient whether she is using, or considering using, CAM. Because the field of CAM is so broad, it is essential to ask which types of CAM thepatient uses. One cannot assume that all alternative therapies are equivalent.

Page 35: TEST 21

Thus, the most appropriate question to ask this patient is "Which alternativemedications do you use?" This is a non-threatening question that will allowher to further detail her use.To ask, "Does your husband know you are using these alternative medications?" (choice A) is inappropriate. The physician'srole is to care for the patient. Whether the patient reveals her use of alternativemedicines to her husband is not the prime concern to the physician. This questionis more likely to create conflict than reveal needed information for the physician.To ask, "Do you realize how dangerous alternative medicines are?" (choice B) is incorrect. This question isconfrontational and judgmental. Many alternative therapies are safe and effective.To ask, "Why don't you stick with traditional medicines?" (choice D) is inappropriate. If a patienthas a condition and there is a remedy from the conventional medical system(known as allopathy in North America) available, then it is reasonable tooffer this remedy as a possibility for the patient. However, inquiring asto why the patient doesn't "stick" with traditional medicine is likely tocause confrontation and a worsening of the patient-doctor relationship.To ask, "Why haven't you revealed your use of alternative medicationsbefore?" (choice E) is also somewhatchallenging and confrontational. Perhaps the patient did not think a conventionalphysician would be accepting of CAM. The important step at this point is toidentify the medications and discuss their risks, benefits, and side effectswith the patient, as one would with traditional medications.

32A 72-year-old man is scheduled to have elective sigmoid resection fordiverticular disease. He has a history of heart disease, and had a documentedmyocardial infarction 2 years ago. He currently does not have angina, buthe lives a sedentary life because "he gets out of breath" if he exerts himself. During the physical examination, it is noted that he has jugular venous distention.He has a hemoglobin of 12 g/dL. If surgery is indeed needed, which of the following should mostlikely be done prior to the operation?

  A. Evaluatethe patient as a candidate for coronary revascularization

  B. Place thepatient on intensive respiratory therapy

  C. Order atransfusion to increase the patient's hemoglobin to 14 g/dL

Page 36: TEST 21

  D. Treat thepatient for congestive heart failure

  E. If at allpossible, wait 6 months before performing surgeryExplanation:The correct answer is D. Jugularvenous distention in this setting is indicative of congestive heart failure,a condition that would make elective surgery very risky. Medical treatmentfor congestive heart failure can reduce the risk.Coronary revascularization (choice A) is a preoperative consideration for patients with progressive angina.Preoperative respiratory therapy (choiceB) is needed for patients with chronic obstructive pulmonary disease(COPD) who have compromised ventilation.A hemoglobin of 12 g/dL should be sufficient, and bringing it up to14 g/dL (choice C) by transfusionswould aggravate the existing congestive failure.Waiting 6 months (choice E)is imperative before noncardiac surgery is done after a myocardial infarction.In this case, the infarction happened 2 years ago; thus, the waiting periodhas already taken place.33A 72-year-old African American man with a history of renal dysfunction,congestive heart failure, and previous myocardial infarction is currentlyundergoing dialysis. Until the past few weeks, he has been in good spiritsand has a strong family support system that helps him in getting to and fromdialysis daily. Over the past few weeks, however, he has been feeling increasinglydepressed and has begun to act bizarrely, with persecutory delusions thatthe government is poisoning the chemicals used on him for dialysis. Also inthe last few weeks, due to increased stomach pain, his medications have beenadjusted and now include cimetidine for stomach ulcers, digoxin, and a babyaspirin daily. He also takes docusate sodium as needed for stool softeningand ibuprofen as needed for mild arthritis pain. Which of the following medications would be most likely to haveinduced the symptoms the patient is now experiencing?

  A. Aspirin

  B. Cimetidine

Page 37: TEST 21

  C. Digoxin

  D. Docusatesodium

  E. IbuprofenExplanation:The correct answer is B. Cimetidineis the only drug listed known to cause psychiatric effects of clinical significance,including delusions and psychosis. The exact mechanism is unknown, but is thought to be related to the effects of cimetidine on the H-2 histamine receptorin the brain. The treatment of cimetidine-induced psychosis is to reduce thedosage of the medication, and discontinue it if an alternative therapy isavailable.Aspirin (choice A) has notbeen shown to induce any clinically significant psychiatric changes.Digoxin (choice C) has beenknown to cause delirium in toxic levels, but does not cause the long standingdelusion that the patient above demonstrates.Docusate sodium (choice D)and ibuprofen (choice E) are medicationsnot known to cause demonstrable psychiatric effects.34A 12-year-old girl comes to the adolescent health clinic because ofa vaginal discharge. The discharge started about 2 months ago and is whitishin color. There is no odor. The patient has no complaints of itching, burning,or pain. The patient started breast development at 9 years of age and herpubertal development has proceeded normally to this point. She has not hadher first menses and she is not sexually active. She has no medical problems.Examination is normal for a 12-year-old female. Microscopic examination ofthe discharge shows no evidence of pseudohyphae, granular-appearing vaginalepithelial cells covered with coccobacillary organisms, or motile organisms. Which of the following is the most likely diagnosis?

  A. Bacterial vaginosis

  B. Candida vulvovaginitis

  C. Physiologic leukorrhea

Page 38: TEST 21

  D. Syphilis

  E. TrichomoniasisExplanation:The correct answer is C. Physiologicleukorrhea can be seen during 2 different periods of childhood. Some femaleneonates develop a physiologic leukorrhea shortly after birth as maternal circulating estrogens stimulate the newborn's endocervical glands and vaginalepithelium. The discharge in these neonates is often gray and gelatinous.Physiologic leukorrhea can also be seen during the months preceding menarche.During this time, rising estrogen levels lead to a whitish discharge not associatedwith any symptoms of irritation. This patient has a whitish discharge, no other symptoms, and she has had normal pubertal development up to this point.The discharge itself has no characteristics of infection. Therefore, physiologicleukorrhea is the most likely diagnosis.Bacterial vaginosis (choice A) isnot the most likely diagnosis in this patient because the discharge is notmalodorous and there are no clue cells seen on microscopic examination ofthe discharge.Candida vulvovaginitis (choiceB) is not the most likely diagnosis because the discharge is notthick and white (or "cottage-cheese"-like) and the patient has no irritativesymptomatology.Syphilis (choice D) most oftenpresents with a painless ulcer (called a chancre) or is found with serologictesting. A nonmalodorous, whitish vaginal discharge in a 12-year-old femalewho is not sexually active is almost certainly not evidence of syphilis.Trichomoniasis (choice E) isalso highly unlikely in this patient and the lack of motile organisms on themicroscopic examination effectively rules out this diagnosis.35A 62-year-old woman has a 4-cm, hard mass under the nipple and areolaof her rather small left breast. The mass occupies most of the breast, butthe breast is freely movable from the chest wall. There is no dimpling orulceration of the skin over the mass, and careful palpation of the axillais completely negative. A core biopsy of the breast mass has established adiagnosis of infiltrating ductal carcinoma, and the mammogram showed no otherlesions in that breast or the other one. A chest x-ray film and liver functiontests are normal. She has no symptoms suggestive of brain or bone metastasis. Which of the following should be offered to this woman

  A. Lumpectomyonly

Page 39: TEST 21

  B. Lumpectomywith axillary sampling and post-op radiation

  C. Total mastectomyonly

  D. Modifiedradical mastectomy (including axillary sampling)

  E. Radicalmastectomy (including complete axillary dissection)Explanation:The correct answer is D. Themass is too large, in a breast that is too small, to allow an adequate lumpectomy.Local control of the tumor requires mastectomy. However, mastectomy aloneis not sufficient. We need to know what is happening in the axilla to makea decision regarding postoperative systemic therapy. The physical examinationwas negative, but the reliability of that finding is not much better thanflipping a coin.Lumpectomy alone (choice A)cannot be done here, and it would not be the appropriate management even ifthe cancer had been smaller.Lumpectomy, axillary sampling, and post-op radiation (choice B) would have been the correct answer for a smaller tumorin a larger breast.Total mastectomy alone (choice C)would have been a wise palliative choice if she had had evidence of distantmetastasis. She does not. We have to go for cure. We need the standard potentiallycurative operation: modified radical mastectomy, which includes axillary sampling.Radical mastectomy (choice E)is no longer used for the treatment of breast cancer. It offers no survivaladvantage over the less mutilating modified radical.36A woman comes to the clinic with her 13-year-old grandson, stating hehas had behavioral problems for the past 4 years. She says that she has beentrying to handle this issue on her own, but it has gotten completely out ofcontrol. Her grandson is living with her now because his mother is in drug rehabilitation center. She describes that he has run from home twice and hasbeen kicked off the school bus for threatening other kids and challengingthe driver. He is about to be expelled from school, since he was caught extortingmoney from other children. Which of the following is the most likely diagnosis?

Page 40: TEST 21

  A. Attention deficit/hyperactivity disorder

  B. Conduct disorder

  C. Intermittent explosive disorder

  D. Oppositional defiant disorder

  E. Rett syndromeExplanation:The correct answer is B. Conductdisorder is a childhood/adolescent disorder defined as a pattern of behaviorin which the basic rights of others are violated with three or more of thefollowing present in the past 12 months: destruction of property, crueltyto animals and people, deceitfulness or theft, and serious violations of rules.It causes clinically significant impairment in social functioning and it is reserved for patients younger than 18.Attention deficit/hyperactivity disorder (choiceA) requires the presence of six symptoms of inattention for atleast 6 months to a degree that is maladaptive and six symptoms of hyperactivity/impulsivitythat cause social impairment. Symptoms are present in two or more settings (e.g., home and school), and some of the symptoms are present before age 7.The symptoms are not due to a general medical condition or other mental disorder.Intermittent explosive disorder (choiceC) is diagnosed in adults only after several episodes of failureto resist aggressive impulses that lead to assaults or destruction of property.The degree of episodes is not proportionate to precipitating stressor. Thedisorder is not due to any other mental disorder or general medical condition.Oppositional defiant disorder (choiceD) is a pattern of negativistic and defiant behavior lasting at least 6 months with four or more of the following: loss of temper, argumentswith adults, defying rules, deliberately annoying other people, blaming othersfor own faults, presence of vindictive behavior, presence of anger, and resentment.Rett syndrome (choice E) belongsto the pervasive developmental disorders. After a normal development, theonset of disorder is between 5 and 48 months and is characterized by deceleration of head growth, loss of previously acquired purposeful hand skills, loss ofsocial engagement, poor coordination of movements, motor mannerisms, and preoccupation

Page 41: TEST 21

with parts or objects. There is no delay in language or cognitive development.37A 29-year-old woman, gravida 3, para 2, at 38 weeks' gestation comesto the physician for a prenatal visit. The patient had hyperemesis in thefirst trimester, but her pregnancy has been otherwise uncomplicated. Her obstetrichistory is significant for a normal spontaneous vaginal delivery of a 9-pound,13-ounce female 5 years ago, and a normal spontaneous vaginal delivery ofa 10-pound, 5-ounce male 3 years ago. She has no medical problems and hasnever had surgery. She takes prenatal vitamins and has no known drug allergies.She states that she feels good fetal movement and has had no contractions,bleeding from the vagina, or loss of fluid. The fetal heart rate is in the 150s/min. Her fundal height is 42 cm and the fetus feels large on examination.The patient states that this fetus feels bigger than her other two. Her cervixis long, closed, and posterior in location. An ultrasound is obtained thatshows a vertex fetus with an estimated fetal weight of 5,100 g. Which of the following is the most appropriate next step in management?

  A. Continueexpectant management

  B. Offer thepatient a cesarean delivery

  C. Offer thepatient an induction of labor

  D. Plan a "cardiacdelivery" for the patient

  E. Requirethat the patient have a cesarean deliveryExplanation:The correct answer is B. Afetus is considered to be macrosomic when its estimated fetal weight exceeds4,500 g. Approximately 1.5% of all fetuses fall into this category. Womenwith macrosomic fetuses are at increased risk of cesarean delivery, postpartumhemorrhage, and vaginal lacerations. The most feared risk, however, is shoulderdystocia, a condition in which the fetal head is delivered but there is adelay in the delivery of the fetal shoulders. Shoulder dystocia can resultin fracture of the clavicle, brachial plexus injury, permanent paralysis,or death. Fetuses weighing more than 4,500 g have a 10 to 25% risk of a shoulder

Page 42: TEST 21

dystocia. Current recommendations by the American College of Obstetriciansand Gynecologists (ACOG, 2000) are that patients with an estimated fetal weightof greater than 5,000 g should be offered a cesarean delivery to prevent shoulderdystocia. This patient has a history of large babies and a current baby withan estimated fetal weight of 5,100 g. She should therefore be offered cesareandelivery.Continuing expectant management (choiceA) would be incorrect because a patient with a fetus of this estimatedweight should, at least, be offered cesarean delivery. If she declines this,continued expectant management is reasonable, but she should be offered theoption of cesarean delivery.Offering the patient an induction of labor (choice C) is incorrect. This patient has an unfavorable cervixand a macrosomic fetus. She has no indication for induction.Planning a "cardiac delivery" for the patient (choice D) is not correct. "Cardiac delivery" refers to the useof forceps to deliver a fetus in a patient who should not push. This typeof delivery is most often done on patients with cardiac problems or certainneurologic conditions. This patient does not require a cardiac delivery.Requiring that the patient have a cesarean delivery (choice E) is incorrect. A patient cannot be forced to have a cesareandelivery. Cesarean delivery is a surgery that requires patient consent. If this patient does not wish to have a cesarean delivery, she is not obligatedto have one.38A 44-year-old woman comes to the office because of a severe headachethat has been present for several weeks and has not responded to the usualover-the-counter headache remedies. She locates the headache to the centerof her head. It is pretty much constant but is worse in the mornings. She has no other neurologic signs or symptoms. She has had "tension headaches"in the past, but she says that those were located in the back of her headand felt different from the present pain. She is currently under considerablestress because she has been unemployed since undergoing modified radical mastectomyfor T3, N1, M0 breast cancer 2 years ago. She had several courses of post-operativechemotherapy, which she eventually discontinued because of the side effects.She has no other complaints at this time. Which of the following is the most appropriate next step in diagnosis?

  A. CT scan of the head

  B. Psychiatric evaluation

  C. Skull x-rays

Page 43: TEST 21

  D. Aortic arch arteriogram

  E. Lumbar punctureExplanation:The correct answer is A. Persistentheadache in a patient with recent history of breast cancer (particularly node-positive)is brain metastasis until proven otherwise. The only acceptable course of action is to take a look (radiologically, of course), and the cheapest andmost reliable way would be a CT scan. For primary brain tumors, the MRI isfavored; however, to show the presence of metastasis, an MRI is not needed.Despite the history of tension headaches and current job worries, psychiatriccauses (choice B) are far down theline in the differential diagnosis.Skull x-rays (choice C) arealmost a vanishing test. They may still show linear skull fractures when youare looking for them, but they would certainly not show intracranial masses.Vascular studies (choice D) were,at one time, the only reliable way to rule out intracranial tumor, but theCT scan displaced them. We still use them to define vascular lesions, whichthis vignette does not suggest.Lumbar puncture (choice E) wouldnot diagnose a tumor and would subject the patient to the risk of herniationof the brainstem. Save that test for patients with meningitis.39A 42-year-old woman is admitted for outpatient elective cosmetic surgery.After the surgery, she develops acute shortness of breath, and pulmonary embolusis diagnosed. Two days later, an emergency psychiatric consultation is called,as the patient has developed a shaking tremor, is pulling out her IV lines,and appears to be watching snakes crawl around the floor of her room and ants crawling on her skin. Which of the following aspects of the patient's history would bemost likely to point to the diagnosis?

  A. Alcoholuse

  B. Depression

  C. Past LSDuse

Page 44: TEST 21

  D. Schizophrenia

  E. TraumaticchildhoodExplanation:The correct answer is A. Thepatient's hallucinations, agitation, and tremor point to the diagnosis of delirium tremens (DTs), which should always be considered when a patient doesnot have access to alcohol after admission to the hospital.Depression (choice B) is nota cause of tremor, although it can be associated with agitation in depressionwith psychotic features.Past LSD use (choice C) hasbeen reported to lead to flashback visual hallucinations, but would not belikely to cause the DTs this patient is experiencing.Schizophrenia (choice D) isa disorder of thought often characterized by auditory hallucinations. It is not associated with formication, i.e., the feeling that insects are crawlingon one's skin.Childhood trauma (choice E)has no demonstrated link to the syndrome exhibited by this patient.40A 34-year-old woman comes the physician because of lower abdominal cramping.The cramping started 2 days ago. Examination is unremarkable except for apelvic examination that reveals a 10-week sized uterus. Urine hCG is positive,and pelvic ultrasound reveals a 10-week intrauterine pregnancy with a fetalheart rate of 160. The patient states that she is not sure whether to keepthe pregnancy. Which of the following is the most appropriate next step in management?

  A. Counselthe patient or refer to an appropriate counselor

  B. Notify thepatient's parents

  C. Notify thepatient's partner

  D. Schedulea termination of pregnancy

Page 45: TEST 21

  E. Tell thepatient that she is likely to have a miscarriageExplanation:The correct answer is A. Thedecision of whether to have a termination of pregnancy is a deeply personalone. This patient has just been notified that she is pregnant with a 10-weekfetus. She is unsure whether she wants to keep her pregnancy or terminateit. In this setting, the most appropriate next step is to counsel the patientregarding her options or refer the patient for counseling. In a balanced way,the patient should be fully informed of all of her options including raisingthe child herself, placing the child up for adoption, and abortion.To notify the patient's parents (choiceB) is not appropriate. Such an act would violate the patient's confidentiality. A 34-year-old woman is an adult and issues of parental notificationdo not apply.To notify the patient's partner (choiceC) is not appropriate. This notification would also violate confidentiality.To schedule a termination of pregnancy (choice D) would not be appropriate. This patient has just informedthe physician that she is unsure what she wants to do. To just go ahead andschedule the termination without proper counseling of the patient would notbe a balanced or proper approach for the patient.To tell the patient that she is likely to have a miscarriage (choice E) is inappropriate. This patient may have a miscarriage, as might any patient with a first-trimester pregnancy.However, once an intrauterine pregnancy with fetal cardiac activity is identified,the risk of miscarriage is approximately 10%. Therefore, she is most likelynot to have a miscarriage.41A 56-year-old alcoholic man gives a history of several years of constantepigastric pain, radiating straight through to the back. The pain is severe,present at all times, and exacerbated by eating. He also has steatorrhea anddiabetes. He relates that he began to have episodes of acute alcoholic pancreatitisin his mid-thirties. At first these would occur every few years, but eventuallyhe would have several attacks per year, until he developed the constant painthat he now has. Although he made numerous attempts to quit drinking he wasnot successful until about 1 year ago, but his current abstinence has notalleviated his pain. Which of the following is the most appropriate initial step in evaluation?

  A. Arteriogram

  B. CT scan

Page 46: TEST 21

of the upper abdomen

  C. Endoscopicretrograde cholangiopancreatogram (ERCP)

  D. Sonogramof the upper abdomen

  E. Upper gastrointestinalseries with bariumExplanation:The correct answer is B. Theclinical diagnosis is chronic pancreatitis, and CT scan will give useful informationabout potentially correctable features of the disease, such as pseudocysts,dilated ducts, calculi, or areas amenable to resection. If surgery is contemplated,ERCP will then be needed. Arteriogram (choice A) addslittle to the diagnosis of chronic pancreatitis, and it is invasive and expensive.

ERCP (choice C) is neededprior to surgery, but it would not precede CT scan in the initial evaluation.

Sonogram (choice D) lacksthe necessary detail for this evaluation. It is an excellent way to followthe size of pseudocysts that are being observed. Upper gastrointestinal series (choiceE) would give minimal information about the pancreas.42An 18-year-old girl with no known past medical history comes to theuniversity health center complaining of fatigue. She says that she feels asif she has barely enough energy to finish her classwork, has been having tensionheadaches, and has been feeling somewhat anxious about how these problemsmay affect her academic progress. Physical examination and laboratory studies, including complete blood count, thyroid studies, serum chemistries, and testingfor mononucleosis, are all within normal limits. When discussing the resultsof these studies she admits to feeling homesick, and states, "I'm worriedthat I'm not good enough to make it here." Sleep and appetite are normal.Despite these problems, she has made many friends and goes out most weekends.She does not drink or use any illicit drugs. Which of the following is the most likely diagnosis?

  A. Adjustment

Page 47: TEST 21

disorder

  B. Bipolardisorder

  C. Generalizedanxiety disorder

  D. Major depressivedisorder

  E. Panic disorderExplanation:The correct answer is A. Thepatient is experiencing a difficulty adjusting to life in college that is manifested by feelings of sadness and mental and physical symptoms of anxiety.The patient's symptoms include both depressive and anxiety-related symptomsthat do not meet criteria for a major depression or anxiety diagnosis suchas panic disorder or generalized anxiety disorder. The patient has no past or present history to indicate a history ofmania or hypomania. Therefore, bipolar disorder (choice B) is incorrect. The patient has symptoms that can be seen in generalized anxiety disorder (choice C), such as tension headaches. However,the chronicity of symptoms and a history of constant anxiety, poor concentration,and insomnia are not suggestive of a diagnosis of generalized anxiety disorder.

The patient's symptoms do not include neurovegetative symptoms or severityof depressive symptoms to warrant a diagnosis of major depressive disorder.Therefore, major depressive disorder (choiceD) is incorrect. There is no history from the clinical vignette to suggest a problemwith recurrent panic attacks. Therefore, a diagnosis of panic disorder (choice E) is incorrect.43A 31-year-old woman, gravida 1, para 0, at 36-weeks' gestation withtwins comes to the physician for a prenatal visit. The patient has had nocontractions, bleeding from the vagina, or loss of fluid, and the babies aremoving well. An ultrasound that was performed today shows that the presentingfetus is vertex and the non-presenting fetus is breech. Both fetuses are appropriatelygrown and greater than 2000 g. The patient wants to know if she should havea vaginal or cesarean delivery. Which of the following is the proper counseling for this patient?

Page 48: TEST 21

  A. Both vaginaldelivery and cesarean delivery are acceptable.

  B. Cesareandelivery is mandated because the fetuses are > 2000g.

  C. Cesareandelivery is mandated because the second twin is breech.

  D. Vaginaldelivery is mandated because the fetuses are > 2000g.

  E. Vaginaldelivery is mandated because the first twin is vertex.Explanation:The correct answer is A. Modeof delivery with twin gestations is an area that has generated controversyover time. Patients with vertex-vertex twins are generally allowed to havea vaginal delivery. Patients with a presenting twin that is non-vertex aregenerally advised to have a cesarean delivery. Patients with the presentingtwin vertex and the non-presenting twin non-vertex may decide which mode ofdelivery they would prefer. Once the presenting (vertex) twin has delivered,there are essentially 2 options for delivery of the second (non-vertex) twin.The first option is an external cephalic version, in which the head of thesecond twin is guided into the pelvis so that it becomes a vertex presentation.The second option is a breech extraction of the second twin. Breech extraction may be performed so long as there is an adequate pelvis, a fetal weight greaterthan 2,000g, an experienced physician, a flexed fetal head, and availablegeneral anesthesia.To state that cesarean delivery is mandated because the fetuses are> 2000g (choice B) is incorrect.The fact that the fetuses are > 2000g makes a vaginal delivery with a non-vertexsecond twin possible.To state that cesarean delivery is mandated because the second twinis breech (choice C) is incorrect.As explained above, vertex-nonvertex twins may be delivered vaginally so longas certain criteria are met.To state that vaginal delivery is mandated because the fetuses are >2000g (choice D) is incorrect. Vaginaldelivery is possible because the fetuses are > 2000g, but the mother may still

Page 49: TEST 21

choose to have a cesarean delivery.To state that vaginal delivery is mandated because the first twin isvertex (choice E) is incorrect. With the first twin vertex, vaginal delivery is possible, but with a non-vertexsecond twin, cesarean delivery would also be entirely appropriate.44A front-seat passenger in a car involved in a head-on collision relatesthat he hit the dashboard with his knees, however, he is specifically complainingof severe pain in his right hip, rather than knee pain. He lies in the stretcherin the emergency department with the right lower extremity shortened, adducted,and internally rotated. Which of the following is the most likely injury?

  A. Femoralneck fracture

  B. Fractureof the shaft of the femur

  C. Intertrochantericfracture

  D. Posteriordislocation of the hip

  E. Posteriordislocation of the kneeExplanation:The correct answer is D. Themechanism of injury is classic. As the knee hits the dashboard in the sittingposition, the femoral head is driven backward and out of the socket. The positionof the injured extremity is also typical, with the internal rotation producedby the posteriorly dislocated femoral head. This injury is an orthopedic emergencybecause of the tenuous blood supply of the femoral head.Femoral neck (choice A) andintertrochanteric (choice C) fracturesare seen in elderly patients who fall and "hurt their hip." They present witha shortened extremity that is externally rotated.A fractured shaft of the femur (choiceB) would produce pain right there (not in the hip) and would havean obvious deformity where the thigh has an angulation that is clearly abnormal.Posterior dislocation of the knee (choice

Page 50: TEST 21

E) is also an orthopedic emergency because of the potential disruptionof the popliteal artery, but the pain and the deformity would be at the knee.45A 27-year-old man is brought to the emergency department by his parentsfollowing a suicide attempt. He tried to shoot himself in the head but thegun misfired. He has a history of depression and has made several suicideattempts in the past. He was going to shoot himself once before, but his father just happened to come home from work early that day and interrupted him. Hewas hospitalized a couple of years ago after attempting to cut a blood vesselin his wrist. He is very short-tempered and has poor anxiety tolerance. Hehas been noncompliant with his medication despite parental support and guidance. Based on this patient's history, studies will most likely demonstratewhich of the following findings?

  A. Changesin the dopaminergic system

  B. Decreased5-hydroxyindolacetic acid levels in the cerebrospinal fluid

  C. Decreasedlevels of thyroid stimulating hormone in serum

  D. Increasedlevels of platelet monoaminooxidase

  E. Normal findingson electroencephalogramExplanation:The correct answer is B. Lowlevels of 5-hydroxyindolacetic acid have been consistently found in the cerebrospinalfluid of victims of violent suicide attempts. Changes in dopaminergic system (choiceA) are not related to suicide attempts. Decreased levels of thyroid-stimulating hormone in serum (choice C) are seen in hyperthyroidism and are indirectly relatedto some affective symptoms such as anxiety or irritability. They are not relatedto violent suicide attempts.Increased platelet monoaminooxidase levels (choice D) is incorrect, as the opposite is actually found in suicidevictims.

Page 51: TEST 21

Electroencephalogram findings (choiceE) are not normal; rather, they are abnormal in suicide attempters.Few studies also found ventricular enlargement.

46A 32-year-old nulliparous woman at 38 weeks' gestation comes to thelabor and delivery ward with regular painful contractions after a gush offluid two hours ago. Her temperature is 98.6 F (37 C). She is found to havegross rupture of membranes and to have a cervix that is 6 centimeters dilated.The fetus is in breech position. The patient is then brought to the operatingroom for cesarean delivery. Which of the following represents the correct procedure for antibioticadministration?

  A. Administerintravenous antibiotics 30 minutes prior to the procedure

  B. Administerintravenous antibiotics after the cord is clamped

  C. Administerintravenous antibiotics immediately after the procedure

  D. Administerintravenous antibiotics for 24 hours after the procedure

  E. Administeroral antibiotics for 1 week following the procedure.Explanation:The correct answer is B. Oneof the major risk factors for developing postpartum endometritis is cesareandelivery. Therefore, prophylactic antibiotics are recommended in all casesof nonscheduled cesarean delivery (i.e., a cesarean delivery that is not anticipated).This patient is having a cesarean delivery because she is a nulliparous womanin labor with a fetus in the breech position. The fact that her membranesbroke a few hours ago, and that she has dilated to 6 centimeters puts herat even higher risk for postpartum endometritis because of the possible exposurethat has occurred to the vaginal flora. Intravenous antibiotics will helpto prevent infection of the mother if they are given before or after the umbilicalcord is clamped. Therefore, to minimize fetal exposure to the antibiotics,the medication should be given after the umbilical cord is clamped. This patient

Page 52: TEST 21

has no evidence of chorioamnionitis; therefore there is no indication to givethe antibiotics immediately.To administer intravenous antibiotics 30 minutes prior to the procedure (choice A) is proper management in a non-pregnantpatient undergoing, for example, a hysterectomy. In the pregnant patient,administering the antibiotic prior to clamping the umbilical cord resultsin unnecessary fetal exposure.To administer intravenous antibiotics immediately after the procedure (choice C) is incorrect because there is aneedless delay. Once the umbilical cord is clamped, there is no further concern regarding unnecessary fetal exposure and the antibiotic may be given immediately.To administer intravenous antibiotics for 24 hours after the procedure (choice D) is indicated when the patient haschorioamnionitis. This patient, however, has no evidence of being infectedand the antibiotics are being given for prophylaxis.To administer oral antibiotics for 1 week following the procedure (choice E) is unnecessary. Once the singledose is given at the time of cord clamping, there is no need for further treatment.47

The night nurse calls the on-call-physician to evaluate a patient whohas become confused. The patient is a 72-year-old man with diabetes who hada large bowel resection for an acute gastrointestinal bleed earlier the sameday. The patient was stable when the nurse began her shift; however, he hasbecome more confused as the night has progressed. He is now a danger to himself as he is attempting to climb out of bed and will not listen to the floor staff.The patient was given a patient-controlled analgesia (PCA) set-up after surgery.It will take the physician approximately 3 minutes to get to the patient'sbedside. Which of the following is the most important management for thispatient before the physician's arrival at the bedside?

  A. Orderinga chest x-ray

  B. Performinga fingerstick for blood glucose level

  C. Placinghim on supplemental oxygen

  D. Sendingoff serum chemistries

Page 53: TEST 21

  E. Stoppingthe infusion of patient-controlled analgesia (PCA)Explanation:The correct answer is C. Thecauses of postoperative disorientation are many. The most lethal one, however,if not promptly recognized and treated is hypoxia. Unless the story clearlystates that the source of delirium is a metabolic problem, the safest thingto do is to assume the patient is hypoxic and treat accordingly.The most common reason for hypoxia on the first night postoperativelyis atelectasis. This may be seen on chest x-ray (choice A); however, the film does not treat the problem and thispatient needs oxygen now and should not wait for an x-ray to be performed.Altered glucose level (too high or too low) may also explain this patient'sconfusion, and a fingerstick should be obtained as part of the evaluation.It is not the first step to be taken, however (choice B).Serum chemistries (choice D)should also be evaluated, as hyponatremia, hypernatremia, uremia, etc., maycause altered mental status. This can be done after oxygen is started, however.Narcotics are also a common cause of confusion, especially in the elderly.These patients have a much longer rate of metabolism of these medicationsand this must be acknowledged at all times. It may be appropriate to stophis PCA pump (choice E), but notas the first step. Besides, the effect of this move may take a long time toappreciate.48A 22-year-old African American man is admitted to a psychiatric unitfor stabilization of an acute episode of psychosis. He has experienced severalweeks of depressive and psychotic symptoms, and has had periods of anxietyand aggression associated with psychotic symptoms and anger about his involuntarycivil commitment. He has been started on an antidepressant, an antipsychotic,and has required additional medications on several occasions for agitation.On the fifth day of his hospitalization, he develops a sustained muscularcontraction involving his left neck muscles, with associated pain and distress. Which of the following medications is most likely responsible forthis acute reaction?

  A. Fluoxetine

  B. Haloperidol

  C. Lorazepam

Page 54: TEST 21

  D. Olanzapine

  E. SertralineExplanation:The correct answer is B. Thispatient is experiencing neuroleptic-induced acute dystonia. About 10% of patientsexperience dystonia as an adverse effect of antipsychotics, usually withinthe first few days of treatment. Dystonia can involve the neck (torticollis),the jaw, the tongue, the eyes, other specific muscle groups, and the entirebody. Dystonias are most common with high potency conventional antipsychotics,such as haloperidol. Pseudoparkinsonism and akathisia are other forms of acuteextrapyramidal side effects.Fluoxetine (choice A) is aselective serotonin reuptake inhibitor (SSRI) that is used in the treatment of depression, bulimia nervosa, and several primary anxiety disorders. Itis not associated with acute dystonic reactions.Lorazepam (choice C) may havebeen used to control anxiety and agitation in this patient. However, benzodiazepinesare not associated with extrapyramidal side effects.Olanzapine (choice D) is anatypical antipsychotic agent. While these agents (including clozapine, risperidone,olanzapine, and quetiapine) may be associated with extrapyramidal side effects, especially at higher dosages, they carry a much lower risk of such neurologicalside effects.Sertraline (choice E), likefluoxetine, is an SSRI used in the treatment of depression and several anxietydisorders. It is not associated with acute dystonia.49A 22-year-old woman, gravida 4, para 3, at 38 weeks' gestation comesto the labor and delivery ward with a gush of fluid. Sterile speculum examinationreveals a pool of fluid that is nitrazine positive and forms ferns when viewedunder the microscope. The fetal heart rate is in the 150s and reactive. An ultrasound demonstrates that the fetus is in the breech position. A cesareandelivery is performed. During the operation, the physician, who has receivedno recent immunizations, is stuck with a needle that had been used on thepatient. Which of the following is this physician at greatest risk of contracting?

  A. HIV

  B. HepatitisB

Page 55: TEST 21

  C. HepatitisC

  D. Scabies

  E. SyphilisExplanation:The correct answer is B. Studieshave shown that surgeons can readily acquire hepatitis B virus from patients.The risk of acquiring hepatitis B is significantly higher than the risk forHIV, and somewhat higher than the risk for hepatitis C. Thus, it is essentialthat health care workers be immunized against the hepatitis B virus. The immunizationschedule is for administration of the vaccine at 1, 2, and 6 months. The Centersfor Disease Control and Prevention recommends that postvaccination testingfor antibodies be performed to identify an adequate response to the immunization.Individuals who do not demonstrate the formation of antibodies after the immunizations are given should be tested for hepatitis B surface antigen to ensure thatthey haven't already been infected. With immunization, the risk of acquiringhepatitis B from a needle stick injury is significantly lessened.HIV (choice A) can be transmittedthrough needle-stick injury. However, the risk of this transmission is lessthan that of hepatitis B in individuals who have not been immunized.Hepatitis C (choice C) appearsto be more transmissible through needle-stick injury than HIV, but less transmissiblethan hepatitis B. However, because there is no immunization for hepatitisC available yet, and because the infection is so widespread in the population,the risk of transmission is of grave concern.Scabies (choice D) is a skinparasite that is transmitted through physical contact.Syphilis (choice E) is a sexuallytransmitted disease that is most often transmitted through sexual contact.Transmission through needle-stick injury is not a primary route.50A 68-year-old man is hospitalized because of a 24-hour history of fever,chills, and a spreading infection of the lower leg of 3 days' duration. Itstarted as tender swelling and redness on the right foot that subsequentlyspread to involve the lower half of the shin. The patient denies injury orany unusual activity in the prior week. He has a history of heart diseaseand hypertension treated with digoxin, losartan, and hydrochlorothiazide.He is plethoric. His blood pressure is 150/90 mmHg, pulse is 95/min and respirationsare 28/min. He is 5 feet 10 inches tall and weighs 230 pounds. Examination of extremities reveals bilateral 2+ nonpitting edema of the lower legs and

Page 56: TEST 21

well-demarcated, intense, shiny erythema and edema of the distal anteromedialright shin and foot with a distinct, slightly raised proximal border. Theaffected area is very warm to touch. Several tense bullae filled with clearfluid are noted on the dorsum of the foot. The toes are edematous and erythematous,with extensive macerated scale in the fourth and fifth interdigital spacesand deep fissures at the base of the affected toe webs. Intravenous antibioticsare initiated and the infection successfully subsides over a 10-day period. Which of the following is the most important step in preventingfuture recurrences of this disease?

  A. Compressiontherapy

  B. Increasingthe dose of the diuretics

  C. Long-termlow-dose antibiotics

  D. Topicalantifungals and meticulous foot hygiene

  E. Weight reductionExplanation:The correct answer is D. Thispatient has erysipelas, a superficial cellulitis of the skin with marked lymphaticinvolvement, caused by beta-hemolytic group A streptococci. Erysipelas mostcommonly occurs in infants, very young children, and the elderly. In mostcases, the organism gains access by direct inoculation through a break inthe skin, but infrequently, hematogenous infection may occur. It begins withan area of redness and enlarges to form the typical tense, painful, brightred, shiny, brawny infiltrated plaque with a well demarcated, distinct, andslightly raised border (as opposed to deeper cellulitis, which has no distinctborder and is flush with surrounding skin). Penicillin is the treatment ofchoice. In people allergic to penicillin, erythromycin or clindamycin is recommended.In patients with no obvious portal of entry, a hidden site of inoculationmust be sought. Frequently, a long-standing fungal infection of the interdigitalspaces will have fissures at the base of the toe webs that are easily overlooked.Unless this simmering dermatophytosis is treated, the infection is bound torecur. Topical antifungals are applied for up to 2 weeks beyond clinical cure,and meticulous foot hygiene should be exercised to prevent recurrence of the

Page 57: TEST 21

fungal infection. The interdigital spaces must be maintained clean and dry.In patients with edema of the toes, drying the interdigital spaces with ahair dryer and regular application of foot powder are very helpful. Compression therapy (choice A)is imperative in treating chronic vascular insufficiency of the lower extremitieswith or without ulceration. Compressive stockings or elastic wraps are appliedin the morning and removed at nighttime. Although they may help with thispatient's leg edema and indirectly decrease the occlusive environment createdby the edematous toes, they would not aid in the resolution of the fungalinfection without appropriate topical antifungal therapy. Increasing the dose of the diuretics (choice B) may alleviate the leg edema and, similar to the compressiontherapy, decrease the environmental factors contributing to this patient'sfungal infection, but it would not prevent future recurrences of cellulitisif not accompanied by antifungal treatment.Long-term low-dose antibiotics (choiceC) are seldom used except for a few chronic inflammatory disorders,such as acne or sarcoidosis, where their antiinflammatory (as opposed to antibacterial) properties are used. Tetracycline antibiotics are most commonly prescribedthis way. Initially, a full antibacterial dose is administered, and with improvement,tapered to the lowest dose that controls the disease. They have no place inthe prophylaxis of erysipelas. Weight reduction (choice E)would benefit this patient's heart disease, hypertension, and overall health,but it would not influence recurrence of erysipelas if the interdigital skinbarrier is not normalized.